The map $L^infty$ to $(L^1)^*$ is not injective in this measure space?












2












$begingroup$


The question is :




Given $X = {a,b}$, with $M = P(X)$ and $mu({a}) = 1,mu({b}) = infty$, find the dual of $L^1(mu)$. Is it $L^infty(mu)$? (Rudin's Real and Complex Analysis, chapter 6)




For solving this question, I first investigated what $L^1(mu)$ is. For this, we note that every function $f : X to mathbb R$ is measurable, with $int_X f
mathrm d mu = f(a)mu({a}) + f(b) mu({b})$
. Under the convention that $0 cdot infty =0$, we see that $L^1(mu)$ consists of all $f$ with $|f(a)| < infty$ and $f(b) =0$.



In other words, let $f_1$ be the indicator of the set ${a}$. Then $L^1(mu) = {cf_1 : |c| < infty}$.



Now, what is $L^infty(mu)$? It consists of functions whose essential supremum is finite. This means that there exists $M$ such that $mu({x : f(x) > M}) = 0$. By definition of $mu$, this forces $f$ to be bounded. Hence, $L^infty(mu)$ is the set of bounded functions on $X$.



We see that for all $g in L^infty(mu)$, the functional $f to int_X fg$ is bounded on $L^1$. So, there is a linear map from $L^infty(mu)$ to $(L^1)^*$.



That it is onto is clear, since every linear functional $T$ is determined by its action on $f_1$, therefore if $Tf_1 = d$, then the function $g = df_1$ is such that $Tf_1 = int f_1 cdot df_1 = d$. So, these linear functionals coincide on the entire of $L^1$.



However, the map is not injective, since if $g(a) = h(a)$ then $g$ and $h$ produce the same functional on $L^1$, since $int gf_1 = int hf_1 = g(a)$. So if $g(b) neq h(b)$ then they produce the same functional despite being different on a set of non-zero measure. The kernel is equal to every function satisfying $g(a) = 0$. Call it $K$.



Therefore, are we right to conclude that $(L^1)^*$ is isomorphic to $L^infty / K$?(first isomorphism theorem)?



My only issue is that I've never seen before, this map being non-injective. For the Lebesgue measure, and others I've seen it has been injective(i.e. if $g$ is such that $int gf = 0$ for all $f in L^1$ then $g=0$ a.e.), and the image has either been the whole or a part of $L^infty$.










share|cite|improve this question









$endgroup$








  • 1




    $begingroup$
    just a side note: strictly speaking $0cdotinfty=0$ is a convention just in notation but it represent a logical truth in measure theory: any sequence of integrals of simple functions on a set of measure zero is zero, hence it point-wise limit is zero regardless of the values it take
    $endgroup$
    – Masacroso
    Nov 30 '18 at 11:11










  • $begingroup$
    @Masacroso Thank you for your input.
    $endgroup$
    – астон вілла олоф мэллбэрг
    Nov 30 '18 at 11:12








  • 2




    $begingroup$
    $L^{1}$ is one dimensional and so is $(L^{1})^{*}$ but $L^{infty}$ is two dimensional.
    $endgroup$
    – Kavi Rama Murthy
    Nov 30 '18 at 11:30










  • $begingroup$
    @KaviRamaMurthy Thank you. So that confirms my suspicion. I just want you to tell me if the above characterization is correct. This is , therefore an example of the case where the map $L^infty$ to $(L^1)^*$ is not injective, by the rank nullity theorem. Is that right?
    $endgroup$
    – астон вілла олоф мэллбэрг
    Nov 30 '18 at 11:32






  • 1




    $begingroup$
    Yes, you have identified the spaces correctly and the dual of $L^{1}$ is not $L^{infty}$.
    $endgroup$
    – Kavi Rama Murthy
    Nov 30 '18 at 11:43


















2












$begingroup$


The question is :




Given $X = {a,b}$, with $M = P(X)$ and $mu({a}) = 1,mu({b}) = infty$, find the dual of $L^1(mu)$. Is it $L^infty(mu)$? (Rudin's Real and Complex Analysis, chapter 6)




For solving this question, I first investigated what $L^1(mu)$ is. For this, we note that every function $f : X to mathbb R$ is measurable, with $int_X f
mathrm d mu = f(a)mu({a}) + f(b) mu({b})$
. Under the convention that $0 cdot infty =0$, we see that $L^1(mu)$ consists of all $f$ with $|f(a)| < infty$ and $f(b) =0$.



In other words, let $f_1$ be the indicator of the set ${a}$. Then $L^1(mu) = {cf_1 : |c| < infty}$.



Now, what is $L^infty(mu)$? It consists of functions whose essential supremum is finite. This means that there exists $M$ such that $mu({x : f(x) > M}) = 0$. By definition of $mu$, this forces $f$ to be bounded. Hence, $L^infty(mu)$ is the set of bounded functions on $X$.



We see that for all $g in L^infty(mu)$, the functional $f to int_X fg$ is bounded on $L^1$. So, there is a linear map from $L^infty(mu)$ to $(L^1)^*$.



That it is onto is clear, since every linear functional $T$ is determined by its action on $f_1$, therefore if $Tf_1 = d$, then the function $g = df_1$ is such that $Tf_1 = int f_1 cdot df_1 = d$. So, these linear functionals coincide on the entire of $L^1$.



However, the map is not injective, since if $g(a) = h(a)$ then $g$ and $h$ produce the same functional on $L^1$, since $int gf_1 = int hf_1 = g(a)$. So if $g(b) neq h(b)$ then they produce the same functional despite being different on a set of non-zero measure. The kernel is equal to every function satisfying $g(a) = 0$. Call it $K$.



Therefore, are we right to conclude that $(L^1)^*$ is isomorphic to $L^infty / K$?(first isomorphism theorem)?



My only issue is that I've never seen before, this map being non-injective. For the Lebesgue measure, and others I've seen it has been injective(i.e. if $g$ is such that $int gf = 0$ for all $f in L^1$ then $g=0$ a.e.), and the image has either been the whole or a part of $L^infty$.










share|cite|improve this question









$endgroup$








  • 1




    $begingroup$
    just a side note: strictly speaking $0cdotinfty=0$ is a convention just in notation but it represent a logical truth in measure theory: any sequence of integrals of simple functions on a set of measure zero is zero, hence it point-wise limit is zero regardless of the values it take
    $endgroup$
    – Masacroso
    Nov 30 '18 at 11:11










  • $begingroup$
    @Masacroso Thank you for your input.
    $endgroup$
    – астон вілла олоф мэллбэрг
    Nov 30 '18 at 11:12








  • 2




    $begingroup$
    $L^{1}$ is one dimensional and so is $(L^{1})^{*}$ but $L^{infty}$ is two dimensional.
    $endgroup$
    – Kavi Rama Murthy
    Nov 30 '18 at 11:30










  • $begingroup$
    @KaviRamaMurthy Thank you. So that confirms my suspicion. I just want you to tell me if the above characterization is correct. This is , therefore an example of the case where the map $L^infty$ to $(L^1)^*$ is not injective, by the rank nullity theorem. Is that right?
    $endgroup$
    – астон вілла олоф мэллбэрг
    Nov 30 '18 at 11:32






  • 1




    $begingroup$
    Yes, you have identified the spaces correctly and the dual of $L^{1}$ is not $L^{infty}$.
    $endgroup$
    – Kavi Rama Murthy
    Nov 30 '18 at 11:43
















2












2








2





$begingroup$


The question is :




Given $X = {a,b}$, with $M = P(X)$ and $mu({a}) = 1,mu({b}) = infty$, find the dual of $L^1(mu)$. Is it $L^infty(mu)$? (Rudin's Real and Complex Analysis, chapter 6)




For solving this question, I first investigated what $L^1(mu)$ is. For this, we note that every function $f : X to mathbb R$ is measurable, with $int_X f
mathrm d mu = f(a)mu({a}) + f(b) mu({b})$
. Under the convention that $0 cdot infty =0$, we see that $L^1(mu)$ consists of all $f$ with $|f(a)| < infty$ and $f(b) =0$.



In other words, let $f_1$ be the indicator of the set ${a}$. Then $L^1(mu) = {cf_1 : |c| < infty}$.



Now, what is $L^infty(mu)$? It consists of functions whose essential supremum is finite. This means that there exists $M$ such that $mu({x : f(x) > M}) = 0$. By definition of $mu$, this forces $f$ to be bounded. Hence, $L^infty(mu)$ is the set of bounded functions on $X$.



We see that for all $g in L^infty(mu)$, the functional $f to int_X fg$ is bounded on $L^1$. So, there is a linear map from $L^infty(mu)$ to $(L^1)^*$.



That it is onto is clear, since every linear functional $T$ is determined by its action on $f_1$, therefore if $Tf_1 = d$, then the function $g = df_1$ is such that $Tf_1 = int f_1 cdot df_1 = d$. So, these linear functionals coincide on the entire of $L^1$.



However, the map is not injective, since if $g(a) = h(a)$ then $g$ and $h$ produce the same functional on $L^1$, since $int gf_1 = int hf_1 = g(a)$. So if $g(b) neq h(b)$ then they produce the same functional despite being different on a set of non-zero measure. The kernel is equal to every function satisfying $g(a) = 0$. Call it $K$.



Therefore, are we right to conclude that $(L^1)^*$ is isomorphic to $L^infty / K$?(first isomorphism theorem)?



My only issue is that I've never seen before, this map being non-injective. For the Lebesgue measure, and others I've seen it has been injective(i.e. if $g$ is such that $int gf = 0$ for all $f in L^1$ then $g=0$ a.e.), and the image has either been the whole or a part of $L^infty$.










share|cite|improve this question









$endgroup$




The question is :




Given $X = {a,b}$, with $M = P(X)$ and $mu({a}) = 1,mu({b}) = infty$, find the dual of $L^1(mu)$. Is it $L^infty(mu)$? (Rudin's Real and Complex Analysis, chapter 6)




For solving this question, I first investigated what $L^1(mu)$ is. For this, we note that every function $f : X to mathbb R$ is measurable, with $int_X f
mathrm d mu = f(a)mu({a}) + f(b) mu({b})$
. Under the convention that $0 cdot infty =0$, we see that $L^1(mu)$ consists of all $f$ with $|f(a)| < infty$ and $f(b) =0$.



In other words, let $f_1$ be the indicator of the set ${a}$. Then $L^1(mu) = {cf_1 : |c| < infty}$.



Now, what is $L^infty(mu)$? It consists of functions whose essential supremum is finite. This means that there exists $M$ such that $mu({x : f(x) > M}) = 0$. By definition of $mu$, this forces $f$ to be bounded. Hence, $L^infty(mu)$ is the set of bounded functions on $X$.



We see that for all $g in L^infty(mu)$, the functional $f to int_X fg$ is bounded on $L^1$. So, there is a linear map from $L^infty(mu)$ to $(L^1)^*$.



That it is onto is clear, since every linear functional $T$ is determined by its action on $f_1$, therefore if $Tf_1 = d$, then the function $g = df_1$ is such that $Tf_1 = int f_1 cdot df_1 = d$. So, these linear functionals coincide on the entire of $L^1$.



However, the map is not injective, since if $g(a) = h(a)$ then $g$ and $h$ produce the same functional on $L^1$, since $int gf_1 = int hf_1 = g(a)$. So if $g(b) neq h(b)$ then they produce the same functional despite being different on a set of non-zero measure. The kernel is equal to every function satisfying $g(a) = 0$. Call it $K$.



Therefore, are we right to conclude that $(L^1)^*$ is isomorphic to $L^infty / K$?(first isomorphism theorem)?



My only issue is that I've never seen before, this map being non-injective. For the Lebesgue measure, and others I've seen it has been injective(i.e. if $g$ is such that $int gf = 0$ for all $f in L^1$ then $g=0$ a.e.), and the image has either been the whole or a part of $L^infty$.







measure-theory lp-spaces






share|cite|improve this question













share|cite|improve this question











share|cite|improve this question




share|cite|improve this question










asked Nov 30 '18 at 11:04









астон вілла олоф мэллбэргастон вілла олоф мэллбэрг

37.4k33376




37.4k33376








  • 1




    $begingroup$
    just a side note: strictly speaking $0cdotinfty=0$ is a convention just in notation but it represent a logical truth in measure theory: any sequence of integrals of simple functions on a set of measure zero is zero, hence it point-wise limit is zero regardless of the values it take
    $endgroup$
    – Masacroso
    Nov 30 '18 at 11:11










  • $begingroup$
    @Masacroso Thank you for your input.
    $endgroup$
    – астон вілла олоф мэллбэрг
    Nov 30 '18 at 11:12








  • 2




    $begingroup$
    $L^{1}$ is one dimensional and so is $(L^{1})^{*}$ but $L^{infty}$ is two dimensional.
    $endgroup$
    – Kavi Rama Murthy
    Nov 30 '18 at 11:30










  • $begingroup$
    @KaviRamaMurthy Thank you. So that confirms my suspicion. I just want you to tell me if the above characterization is correct. This is , therefore an example of the case where the map $L^infty$ to $(L^1)^*$ is not injective, by the rank nullity theorem. Is that right?
    $endgroup$
    – астон вілла олоф мэллбэрг
    Nov 30 '18 at 11:32






  • 1




    $begingroup$
    Yes, you have identified the spaces correctly and the dual of $L^{1}$ is not $L^{infty}$.
    $endgroup$
    – Kavi Rama Murthy
    Nov 30 '18 at 11:43
















  • 1




    $begingroup$
    just a side note: strictly speaking $0cdotinfty=0$ is a convention just in notation but it represent a logical truth in measure theory: any sequence of integrals of simple functions on a set of measure zero is zero, hence it point-wise limit is zero regardless of the values it take
    $endgroup$
    – Masacroso
    Nov 30 '18 at 11:11










  • $begingroup$
    @Masacroso Thank you for your input.
    $endgroup$
    – астон вілла олоф мэллбэрг
    Nov 30 '18 at 11:12








  • 2




    $begingroup$
    $L^{1}$ is one dimensional and so is $(L^{1})^{*}$ but $L^{infty}$ is two dimensional.
    $endgroup$
    – Kavi Rama Murthy
    Nov 30 '18 at 11:30










  • $begingroup$
    @KaviRamaMurthy Thank you. So that confirms my suspicion. I just want you to tell me if the above characterization is correct. This is , therefore an example of the case where the map $L^infty$ to $(L^1)^*$ is not injective, by the rank nullity theorem. Is that right?
    $endgroup$
    – астон вілла олоф мэллбэрг
    Nov 30 '18 at 11:32






  • 1




    $begingroup$
    Yes, you have identified the spaces correctly and the dual of $L^{1}$ is not $L^{infty}$.
    $endgroup$
    – Kavi Rama Murthy
    Nov 30 '18 at 11:43










1




1




$begingroup$
just a side note: strictly speaking $0cdotinfty=0$ is a convention just in notation but it represent a logical truth in measure theory: any sequence of integrals of simple functions on a set of measure zero is zero, hence it point-wise limit is zero regardless of the values it take
$endgroup$
– Masacroso
Nov 30 '18 at 11:11




$begingroup$
just a side note: strictly speaking $0cdotinfty=0$ is a convention just in notation but it represent a logical truth in measure theory: any sequence of integrals of simple functions on a set of measure zero is zero, hence it point-wise limit is zero regardless of the values it take
$endgroup$
– Masacroso
Nov 30 '18 at 11:11












$begingroup$
@Masacroso Thank you for your input.
$endgroup$
– астон вілла олоф мэллбэрг
Nov 30 '18 at 11:12






$begingroup$
@Masacroso Thank you for your input.
$endgroup$
– астон вілла олоф мэллбэрг
Nov 30 '18 at 11:12






2




2




$begingroup$
$L^{1}$ is one dimensional and so is $(L^{1})^{*}$ but $L^{infty}$ is two dimensional.
$endgroup$
– Kavi Rama Murthy
Nov 30 '18 at 11:30




$begingroup$
$L^{1}$ is one dimensional and so is $(L^{1})^{*}$ but $L^{infty}$ is two dimensional.
$endgroup$
– Kavi Rama Murthy
Nov 30 '18 at 11:30












$begingroup$
@KaviRamaMurthy Thank you. So that confirms my suspicion. I just want you to tell me if the above characterization is correct. This is , therefore an example of the case where the map $L^infty$ to $(L^1)^*$ is not injective, by the rank nullity theorem. Is that right?
$endgroup$
– астон вілла олоф мэллбэрг
Nov 30 '18 at 11:32




$begingroup$
@KaviRamaMurthy Thank you. So that confirms my suspicion. I just want you to tell me if the above characterization is correct. This is , therefore an example of the case where the map $L^infty$ to $(L^1)^*$ is not injective, by the rank nullity theorem. Is that right?
$endgroup$
– астон вілла олоф мэллбэрг
Nov 30 '18 at 11:32




1




1




$begingroup$
Yes, you have identified the spaces correctly and the dual of $L^{1}$ is not $L^{infty}$.
$endgroup$
– Kavi Rama Murthy
Nov 30 '18 at 11:43






$begingroup$
Yes, you have identified the spaces correctly and the dual of $L^{1}$ is not $L^{infty}$.
$endgroup$
– Kavi Rama Murthy
Nov 30 '18 at 11:43












0






active

oldest

votes











Your Answer





StackExchange.ifUsing("editor", function () {
return StackExchange.using("mathjaxEditing", function () {
StackExchange.MarkdownEditor.creationCallbacks.add(function (editor, postfix) {
StackExchange.mathjaxEditing.prepareWmdForMathJax(editor, postfix, [["$", "$"], ["\\(","\\)"]]);
});
});
}, "mathjax-editing");

StackExchange.ready(function() {
var channelOptions = {
tags: "".split(" "),
id: "69"
};
initTagRenderer("".split(" "), "".split(" "), channelOptions);

StackExchange.using("externalEditor", function() {
// Have to fire editor after snippets, if snippets enabled
if (StackExchange.settings.snippets.snippetsEnabled) {
StackExchange.using("snippets", function() {
createEditor();
});
}
else {
createEditor();
}
});

function createEditor() {
StackExchange.prepareEditor({
heartbeatType: 'answer',
autoActivateHeartbeat: false,
convertImagesToLinks: true,
noModals: true,
showLowRepImageUploadWarning: true,
reputationToPostImages: 10,
bindNavPrevention: true,
postfix: "",
imageUploader: {
brandingHtml: "Powered by u003ca class="icon-imgur-white" href="https://imgur.com/"u003eu003c/au003e",
contentPolicyHtml: "User contributions licensed under u003ca href="https://creativecommons.org/licenses/by-sa/3.0/"u003ecc by-sa 3.0 with attribution requiredu003c/au003e u003ca href="https://stackoverflow.com/legal/content-policy"u003e(content policy)u003c/au003e",
allowUrls: true
},
noCode: true, onDemand: true,
discardSelector: ".discard-answer"
,immediatelyShowMarkdownHelp:true
});


}
});














draft saved

draft discarded


















StackExchange.ready(
function () {
StackExchange.openid.initPostLogin('.new-post-login', 'https%3a%2f%2fmath.stackexchange.com%2fquestions%2f3019958%2fthe-map-l-infty-to-l1-is-not-injective-in-this-measure-space%23new-answer', 'question_page');
}
);

Post as a guest















Required, but never shown

























0






active

oldest

votes








0






active

oldest

votes









active

oldest

votes






active

oldest

votes
















draft saved

draft discarded




















































Thanks for contributing an answer to Mathematics Stack Exchange!


  • Please be sure to answer the question. Provide details and share your research!

But avoid



  • Asking for help, clarification, or responding to other answers.

  • Making statements based on opinion; back them up with references or personal experience.


Use MathJax to format equations. MathJax reference.


To learn more, see our tips on writing great answers.




draft saved


draft discarded














StackExchange.ready(
function () {
StackExchange.openid.initPostLogin('.new-post-login', 'https%3a%2f%2fmath.stackexchange.com%2fquestions%2f3019958%2fthe-map-l-infty-to-l1-is-not-injective-in-this-measure-space%23new-answer', 'question_page');
}
);

Post as a guest















Required, but never shown





















































Required, but never shown














Required, but never shown












Required, but never shown







Required, but never shown

































Required, but never shown














Required, but never shown












Required, but never shown







Required, but never shown







Popular posts from this blog

Le Mesnil-Réaume

Ida-Boy-Ed-Garten

web3.py web3.isConnected() returns false always